SlideShare une entreprise Scribd logo
1  sur  9
Télécharger pour lire hors ligne
C ONTAGENS D UPLAS
                                                     S AMUEL F EITOSA


                    ´                                 ´
    Em um grande numero de problemas de combinatoria precisamos contar algum conjunto de pelo menos duas maneiras
                      ´
diferentes. Nosso proposito ser´ estudar algumas formas de fazer tais contagens. O leitor deve PENSAR bastante em cada
                               a
                            ¸˜        ¸˜                                                                ´
problema antes de ver a solucao. As secoes est˜ o organizadas em ordem crescente de dificuldade, sendo a ultima bem mais
                                              a
avancada que as demais.
     ¸


1 Grafos
                        a                                                    ´
Problema 1. Em Brasilˆ ndia existem apenas 9 casas muito distantes entre si. E poss´vel que cada casa esteja ligada a exata-
                                                                                   ı
mente 7 outras casas atrav´ s de estradas?
                          e

Solucao N˜ o e poss´vel. Some a quantidade de estradas que saem de cada casa. Bem, facilmente obtemos 7 × 9 = 63
     ¸˜     a ´       ı
                                                                                                             ´
estradas. Como cada estrada liga duas cidades, a contagem que fizemos contou cada estrada duas vezes. Logo o numero
obtido teria que ser par.

      a                                                               ´
   Ser´ que cada casa estar ligada a exatamente 7 outras foi crucial? E poss´vel revolvermos o problema anterior com um
                                                                            ı
enunciado mais geral:


Problema 2. Prove que numa festa com n pessoas, o numero de pessoas que conhecem um numero ´mpar de outras pessoas
                                                   ´                                 ´     ı
         ´
na festa e par.

Solucao Numere as pessoas de 1 at´ n e denote por di o numero de amigos da pessoa i. Imagine que existe um fio entre
     ¸˜                           e                     ´
duas pessoas que se conhecem. Se E denota a quantidade de fios, temos

                                                    d1 + d2 + . . . + dn = 2E,

             ´                                                             ´
pois cada fio e contado duas vezes, um para cada ponta. Como o lado direito e par, no lado esquerdo devemos ter uma
quantidade par de numeros ´mpares.
                    ´     ı

                    ´
Para professores. E recomend´ vel que, na sala de aula, o professor tente primeiro introduzir as ideias envolvidas na con-
                                a
                                            ¸˜                                                     ´
tagem dupla anterior sem introduzir notacao desnecess´ ria, por exemplo, ao explicar o que e um grafo antes mesmo de
                                                          a
resolver alguns problemas. Enunciados envolvendo objetos completamente diferentes, pessoas e casas como nos exemplos
                      ´
anteriores, podem ser uteis para levar o aluno a ituir que as ideias s˜ o mais gerais e abstratas.
                                                                      a


Problema 3. Prove que numa festa com 2n, pessoas existem duas com um numero par de amigos em comum.
                                                                      ´

Solucao Suponha que quaisquer duas pessoas tenham um numero ´mpar de amigos em comum e seja A um dos partici-
     ¸˜                                                              ´      ı
pantes da festa. Seja M = {F1 , F2 , . . . , Fk } o conjunto dos amigos de A. Considere uma nova festa restrita apenas ao conjunto
M . Como cada Fi tem um numero ´mpar de amigos em comum com A, na nova festa, cada Fi possui um numero ´mpar de
                            ´           ı                                                                          ´       ı
amigos. Pelo exemplo anterior, k deve ser par. O mesmo argumento vale para qualquer pessoa na festa e consequentemente
todos conhecem um numero par de pessoas. Peca para cada um dos amigos de A fazerem uma lista de seus amigos difer-
                       ´                                ¸
entes de A. A soma da quantidade de nomes listados e par, pois e uma soma de uma quantidade par (igual a k) de numeros
                                                             ´         ´                                                     ´
´mpares (cada Fi possui um numero ´mpar de amigos diferentes de A). Agora comparemos o numero de aparicoes de cada
ı                              ´           ı                                                          ´               ¸˜
uma das 2n−1 pessoas diferentes de A nessas listas. Se cada uma delas aparecer em um numero ´mpar de listas, a soma total
                                                                                              ´       ı
de todos os nomes em todas as listas seria ´mpar (Lembre-se que a soma de uma quantidade ´mpar de numeros ´mpares e
                                                   ı                                                ı            ´       ı       ´
ı                  ´               ¸˜                                                     ´
´mpar!). Mas isso e uma contradicao. Logo, existem duas pessoas na festa com um numero par de amigos em comum.

                                          a        ´
Problema 4. (Lema de Sperner) Um triˆ ngulo e dividido em triˆ ngulos menores de modo que quaisquer dois dentre os
                                                                       a
triˆ ngulos menores ou n˜ o tˆ m ponto em comum, ou tˆ m um v´ rtice em comum, ou tˆ m um lado (completo) em comum.
   a                       a e                             e         e                      e
Os v´ rtices do triˆ ngulo maior s˜ o numerados: 1, 2, 3. Os v´ rtices dos triˆ ngulos menores tamb´ m s˜ o numerados: 1, 2 ou 3.
      e            a              a                           e               a                     e   a
A numeracao e arbitr´ ria, exceto que os v´ rtices sobre o lado do triˆ ngulo maior oposto ao v´ rtice i (i = 1, 2, 3) n˜ o podem
           ¸˜ ´         a                  e                             a                        e                     a


                                                                1
receber o numero i (veja a figura). Mostre que entre os triˆ ngulos menores existe um cujos v´ rtices s˜ o numerados 1, 2, 3.
           ´                                              a                                 e         a

   (Colocar Figura)

       ¸˜
Solucao Uma boa estrat´ gia seria pensar numa vers˜ o particular do problema. Olhando para o bordo do triˆ ngulo, temos
                          e                           a                                                          a
            ¸˜
uma situacao semelhante ao problema inicial com uma dimens˜ o e uma cor a menos. Consideremos, por exemplo, o lado
                                                                 a
dos v´ rtices 1 e 2, poder´amos provar que dentre os segmentos da divis˜ o deste lado, sempre existe um numero ´mpar de
       e                  ı                                               a                                   ´        ı
segmentos com as cores 1 e 2. Imagine uma pessoa com uma bandeira abaixada no v´ rtice 1 caminhando em direcao ao
                                                                                         e                               ¸˜
v´ rtice 2. Ao passar por um v´ rtice vermelho, a pessoa deve levantar a bandeira e ao passar por um v´ rtice branco a pessoa
  e                           e                                                                        e
deve abaixar a bandeira. Ao final do trajeto, a bandeira estar´ abaixada e consequentemente a pessoa ter´ realizado um
                                                                a                                               a
numero ´mpar de movimentos de abaixar e levantar a bandeira. Mas cada movimento de abaixar ou levantar a bandeira
  ´       ı
corresponde a um segmento com as cores 1 e 2. Outro modo de ver isso, e perceber que a adicao de um vertice de qualquer
                                                                           ´                   ¸˜
uma das duas cores, n˜ o altera a paridade da quantidade de segmentos com v´ rtices de cores diferentes. Agora tentemos
                        a                                                        e
usar essa informacao para resolver o problema. Contaremos o numero de segmentos 12 (com algumas repeticoes). Eles
                     ¸˜                                             ´                                                ¸˜
aparecem nos triˆ ngulos de v´ rtices 123, 122 e 112. Digamos que h´ x triˆ ngulos 123, y triˆ ngulos 122 e z triˆ ngulos 112.
                   a           e                                      a      a                a                    a
Observe que os segmentos internos ao triˆ ngulo grande s˜ o contados duas vezes (eles s˜ o comuns a dois triˆ ngulos) e os
                                            a               a                              a                      a
segmentos do lado do triˆ ngulo grande, somente uma vez. Notemos tamb´ m que os segmentos 12 aparecem duas vezes nos
                          a                                                 e
triˆ ngulos 122 e 112 e uma vez nos triˆ ngulos 123. Assim,
   a                                    a

                           2 × segmentos interiores 12 + segmentos nos lados 12 =
                                                       numero de segmentos 12 = x + 2y + 2z.
                                                         ´


   Como existe uma quantidade ´mpar de segmentos nos lados, conclu´mos que x e ´mpar.
                              ı                                   ı          ´ı

Exerc´cios Propostos
     ı
  1. Em uma festa com 23 pessoas, e poss´vel que cada um possua 1, 3 ou 5 amigos na festa?
                                  ´     ı
     ´
  2. E poss´vel desenhar 9 segmentos de reta no plano de tal forma que cada um intersecta exatamente 3 outros?
           ı
                                   a                                   a                                    ´
  3. Existem 1000 cidades em Brazilˆ ndia e alguns pares de cidades s˜ o ligadas por uma estrada de terra. E poss´vel viajar
                                                                                                                 ı
     de uma cidade para qualquer outra atrav´ s das estradas de terra. Prove que o governo de Brazilˆ ndia pode pavimentar
                                             e                                                      a
     algumas estradas de modo que de cada cidade saia um numero ´mpar de estradas pavimentadas.
                                                              ´       ı


2 Identidades Binomiais
                                                         ´             ´         ´
Uma excelente maneira de valorizar o significado de um numero binomial e, logo apos defin´-lo, estudar propriedades deste
                                                                                         ı
                                 a                       ¸                         ´
objeto sem necessariamente calcul´ -lo explicitamente. Facamos o mesmo aqui: O numero de maneiras de selecionarmos
um subconjunto de r elementos distintos de um conjunto de n elementos distintos, onde a ordem da selecao n˜ o importa, e
                                                                                                      ¸˜ a             ´
denotado por n .
              r

                                    ¸˜
Problema 5. Prove as seguintes afirmacoes:
                           n    n            n+1
          ¸˜
  1. (Relacao de Stifel)     +           =       .
                           r   r+1           r+1
       2n + 2          2n    2n    2n
  2.            =         +2    +     .
        n+1           n+1    n    n−1
    ¸˜
Solucao

  1. O lado direito conta o numero de maneiras de escolhermos r + 1 pessoas em um grupo de n + 1. Podemos dividir essas
                             ´
     escolhas em dois grupos: aquelas que cont´ m um certo indiv´duo ( n ) e aquelas que n˜ o o cont´ m ( r+1 ).
                                              e                 ı       r                 a         e      n


  2. O lado esquerdo conta o numero de maneiras de escolhermos n + 1 pessoas no grupo da 2n + 2 pessoas que est˜ o
                                  ´                                                                                    a
     comemorando o aniver´ rio da Ana e do jo˜ o. Essas escolhas se dividem em trˆ s grupos, aquelas que n˜ o cont´ m Ana
                              a                 a                                   e                     a       e
     e Jo˜ o ( n+1 ), aquelas que cont´ m apenas um dos dois (2 2n ) e aquelas que cont´ m ambos ( n−1 ).
         a      2n
                                      e                          n                     e            2n


Problema 6. (Teorema das Colunas) Mostre que:
                                                 k
                                                      n+i         n+k+1
                                                             =          .
                                                i=0
                                                       n           n+1


                                                              2
Solucao Suponha que exista uma fila de n + 1 + k pessoas. As primeiras k + 1 pessoas s˜ o c0 , c1 , . . . , ck e est˜ o ordenadas
     ¸˜                                                                                           a                 a
pelas suas alturas. O lado direito conta o numero de maneiras de escolhermos n + 1 pessoas nesse grupo. Certamente
                                              ´
precisaremos escolher algu´ m do conjunto {c0 , c1 , . . . , ck } = C. Seja Ci o conjunto das escolhas em que o menor elemento de
                           e
C escolhido e ci . Qualquer escolha faz parte de algum desses Ci ’s. Para calcular o numero de elementos de Ci , veja que ci
             ´                                                                               ´
deve fazer parte dessa escolha e os outros n elementos devem ser escolhidos dentre os elementos posteriores a ci , i.e., temos
n + k + 1 − i candidatos. Logo,
                                                                            k                k
                                                      n+k+1                                       n+i
                                                                     =          |Ci | =               .
                                                       n+1               i=0                i=0
                                                                                                   n

                                    n     n             n
Problema 7. Mostre que: 1 ·           +2·   + ... + n ·                         = n2n−1
                                    1     2             n
     ¸˜                      ´
Solucao Vamos contar o numero de maneiras de escolhermos algumas criancas para passearem e, al´ m disso, uma delas
                                                                               ¸                        e
para ganhar um sorvete. Como a crianca que ganhar´ o sorvete certamente estar´ no passeio, podemos comecar escolhendo-
                                       ¸           a                             a                           ¸
a. Podemos fazer isso de n maneiras. Das criancas que restaram, devemos escolher algum subconjunto para acompanhar a
                                                ¸
primeira crianca, podemos fazer isso de 2n−1 maneiras, isso nos d´ o lado direito. Por outro lado, para cada k ≥ 1, podemos
               ¸                                                 a
escolher k criancas ( n ) e, posteriormente, podemos escolher uma delas para ganhar o sorvete de k maneiras. A soma sobre
                 ¸    k
todos os k, contar´ todos os conjuntos.
                   a
Problema 8. (Putnam 1962) Mostre que:
                                                            n
                                                                     n
                                                                r2       = n(n + 1)2n−2 .
                                                         r=1
                                                                     r
     ¸˜                                ¸˜
Solucao Vamos construir uma situacao semelhante ao problema anterior, sendo que agora seremos mais bondosos, al´ m       e
de escolher uma crianca para ganhar um sorvete tamb´ m escolheremos uma crianca para ganhar um caramelo (a crianca
                       ¸                                e                            ¸                                    ¸
                                                                                                                   ´
pode ser a mesma). O lado esquerdo conta essas escolhas como no problema anterior. Agora queremos contar o numero de
maneiras de primeiro escolhermos as criancas que v˜ o receber as guloseimas e depois aquelas que ir˜ o acompanh´ -las. Se
                                            ¸        a                                                  a            a
essas duas criancas s˜ o diferentes, temos n(n − 1)2n−2 escolhas. Se essas criancas s˜ o iguais, temos n2n−1 escolhas. Agora
                ¸    a                                                          ¸    a
basta ver que:
                                           n(n − 1)2n−2 + n2n−1 = n(n + 1)2n−2 .

Exerc´cios Propostos
     ı
  1. Prove as seguntes identidades utilizando argumentos de contagens duplas:
              n   n       n
       (a)      +   + ...                 = 2n
              0   1       n
             n
                           n
       (b)       (−1)k         =0
                           k
             k=0
              n        m   n  n−r
       (c)                 =       .
              m        r   r  m−r
                   n       n−1                        n                                     n!
      (d) k ·          =n·     e com isso conclua que                               =              .
                   k       k−1                        k                                 k!(n − k)!
                                                  k
                                                        n        m                  n+m
       (e) (Identidade de Vandermonde)                                   =              .
                                                 j=0
                                                        j       k−j                  k

  2. Considere a sequˆ ncia de Fibonacci definida por F0 = 0, F1 = 1 e Fn+2 = Fn+1 + Fn . Prove que:
                     e
       (a) O numero de maneiras de cobrirmos um tabuleiro 2 × n, sem sobreposicao, com pecas 1 × 2 e Fn+1 .
              ´                                                               ¸˜         ¸         ´
       (b) Fn+m = Fn−1 Fm + Fn Fm+1 .
                 n   n−k+1
       (c)       k=0   k       = Fn+1 .
  3. Definimos por Dn o numero de permutacoes de {1, 2 . . . , n} sem pontos fixos (um elemento i e um ponto fixo quando
                            ´                ¸˜                                                  ´
     ele ocupa a posicao i na permutacao). Usando o princ´pio da inclus˜ o-exclus˜ o podemos obter:
                     ¸˜              ¸˜                  ı             a         a
                                                                         1  1                1
                                                      Dn = n! 1 −          + − . . . + (−1)n              .
                                                                         1! 2!               n!
     Mostre que:
                                                                                n
                                                                                        n
                                                                     n! =                 Dn−r .
                                                                            r=0
                                                                                        r


                                                                                3
´
  4. Encontre uma formula fechada para a soma
                                                            2                     2                 2
                                                    n                     n                     n
                                                                +                     + ... +           .
                                                    0                     1                     n

  5. Seja n um inteiro n˜ o negativo. Mostre que:
                        a
                                                        n                     2
                                                                     n                   2n − 1
                                                                k                 =n
                                                                     k                    n−1
                                                    k=0



3 Tabuleiros
                                                         ` ´
Se o que queremos contar est´ de alguma forma associado a numeros escritos em um tabuleiro, temos uma contagem dupla
                            a
                                    ´                        ´       `           ´
extremamente natural: a soma dos numeros escritos nas linhas e igual a soma dos numeros escritos nas colunas.

                    ı                                                            ¸˜
Problema 9. (Olimp´ada Russa) Duzentos estudantes participaram de uma competicao matem´ tica. A prova possuia 6
                                                                                            a
problemas. Sabemos que cada problema foi resolvido corretamente por pelo menos 120 estudantes. Prove que existem dois
                                                                                              ¸˜
participantes de modo que para qualquer problema, pelo menos um deles dois conseguiu uma solucao correta.

Solucao Facamos um tabuleiro 200 × 6 representando o resultado dos estudantes. Cada linha representar´ um estudante
      ¸˜     ¸                                                                                             a
e cada problema resolvido pelo estudante i ser´ marcado com o numero 1 na tabela. Caso o problema n˜ o tenha sido re-
                                              a                     ´                                    a
                         ´
solvido, marcaremos o numero zero. Pensemos inicialmente em casos extremais. O que acontece se um estudante resolveu
os seis problemas? Basta escolhermos um estudante qualquer e a dupla desejada estar´ formada. Se um estudante resolveu
                                                                                      a
5 problemas, tamb´ m podemos obter facilmente nossa dupla. E se um estudante tiver resolvido exatamente 4 problemas?
                   e
Suponha, sem perda de generalidade, que ele n˜ o resolveu os problemas 5 e 6. Sabemos que pelo menos 120 pessoas re-
                                               a
solveram o problema 5. Se nenhuma delas tiver resolvido o problema 6, saberemos que no m´ ximo 80 pessoas o resolveram.
                                                                                            a
Mas isso contradiz o enunciado e assim temos certeza que pelo menos uma pessoa resolveu ambos os problemas. Resta
mostrar que esse tipo de situacao sempre acontece, i.e., existe algu´ m que resolveu pelo menos 4 problemas. Agora usare-
                              ¸˜                                    e
mos a contagem dupla. A soma dos numero das colunas e pelo menos 6 × 120 = 720. Como existem 200 linhas, pelo menos
                                      ´                   ´
                     720
uma delas ter´ soma 200 > 3, ou seja, pelo menos uma linha ter´ 4 numeros 1’s.
               a                                                 a    ´

    A estrat´ gia do problema anterior foi associar uma matriz com entradas em {0, 1} que codificasse o enunciado e realizar
            e
                               ´                                                                         ´
as duas somas poss´veis no numero de uns: pelas linhas e pelas colunas. Outra estrat´ gia importante e contar pares de
                      ı                                                                  e
  ´                                                                               ¸˜
numeros de uns ou zeros em uma mesma linha ou coluna. Vejamos uma nova solucao para o problema anterior usando essa
ideia.

     ¸˜                          ¸˜ ´
Solucao Suponha que a afirmacao e falsa, i.e., para cada par de estudantes, existe pelo menos um problema que n˜ o foi
                                                                                                                  a
resolvido por eles. Facamos uma matriz 200 × 6 como anteriormente. Para cada par de linhas j e k, cole uma etiqueta
                      ¸
Ejk ao problema i se ele n˜ o foi resolvido pelos estudantes correspondentes. Contemos o numero de etiquetas utilizadas.
                          a                                                               ´
Para cada par de linhas, devemos usar pelo menos uma etiqueta, logo o numero m´nimo de etiquetas e 200 . Como cada
                                                                          ´        ı                 ´ 2
                                                                                                     80
problema foi resolvido por pelo menos 120 estudantes, os seis problemas podem receber no m´ ximo 6 · 2 etiquetas. Como
                                                                                          a
 200
  2  < 6 · 80 , temos um absurdo.
            2

Problema 10. (Olimp´ada Russa) Bruno pintou k casas de um tabuleiro n × n de preto. Ele observou que n˜ o existiam
                       ı                                                                              a
quatro casas pretas formando um um retˆ ngulo com lados paralelos aos lados do tabuleiro. Mostre que:
                                      a
                                                           √
                                                       1 + 4n − 3
                                                k≤n                   .
                                                            2


     ¸˜
Solucao Ao pintar duas casas pretas em uma mesma linha, Bruno cria um empedimento para sua pintura: n˜ o poder´
                                                                                                              a        a
pintar outras duas casas pretas nessas colunas. Comecemos contando esses empedimentos. Denotemos por ai o numero de
                                                                                                               ´
                                      n
casas pintadas na linha i, ent˜ o i=1 ai = k. Para cada par de casas pintadas na linha i, associemos uma etiqueta Ejk se
                               a
essas casas est˜ o nas colunas j e k. O numero de etiquetas utilizadas e
               a                         ´                             ´
                                                                    n
                                                                              ai
                                                                                 ,
                                                                    i=1
                                                                              2

                           ai
pois em cada linha temos   2    pares de quadrados pintados. Como n˜ o podemos repetir etiquetas, pois assim formar´amos
                                                                   a                                               ı



                                                                          4
n
                ´      a                              ´
um quadrado, o numero m´ ximo de etiquetas utilizadas e                         2   . Logo,
                                                                 n
                                                                          ai                 n
                                                                                        ≤
                                                                i=1
                                                                          2                  2
                                                            n
                                                                 a2 − ai
                                                                  i                         n2 − n
                                                                                        ≤          .
                                                          i=1
                                                                    2                          2
                                  n                    n
Pela desigualdade de Cauchy, (    i=1   a2 )n
                                         i      ≥(     i=1      ai )2 = k 2 , consequentemente,
                                                k2    k       n2 − n
                                                   −      ≤
                                                2n 2n            2
Estudando o sinal da inequacao do segundo grau em k, obtemos que,
                           ¸˜
                                                          √
                                                       1 + 4n − 3
                                               k≤n                   .
                                                            2
Problema 11. (IMO 1998/2) Num concurso, h´ a candidatos e b ju´zes, onde b ≥ 3 e ´mpar. Cada candidato e avaliado por
                                               a                   ı                ´ı                     ´
cada juiz, podendo passar ou n˜ o. Sabe-se que os julgamentos de cada par de ju´zes coincidem em no m´ ximo k candidatos.
                              a                                                ı                     a
Prove que
                                                        k    b−1
                                                          ≥
                                                        a     2b


Solucao Facamos um tabuleiro a × b onde as linhas representam os candidatos e as colunas os ju´zes. Na intersecao de uma
      ¸˜     ¸                                                                                ı               ¸˜
linha e coluna, colocaremos um numero 1 caso aquele ju´z tenha aprovado aquele estudante e o numero 0 caso contr´ rio.
                                   ´                   ı                                         ´                   a
Se os ju´zes das colunas i e j concordam com o julgamento do aluno da coluna k, pregamos uma etiqueta Eij nesse aluno.
         ı
                 ´
Calculemos o numero de etiquetas pregadas de duas maneiras. Primeiramente, para cada par de colunas, sabemos que
               a                                     ´                         ´                 ` b
existem no m´ ximo k etiquetas associadas. Logo, o numero de etiquetas usadas e menor ou igual a 2 k. Para contar esse
numero usando as linhas, denotaremos por ai o numero de zeros na linha i. Assim, para cada linha, temos exatamente
   ´                                               ´
  ai
  2  + b−ai . As duas contagens resultam em:
           2
                                                 a
                                                            ai                 b − ai                     b
                                                                      +                          ≤ k
                                                i=0
                                                            2                    2                        2

Agora vamos fazer outra contagem dupla para estimar o termo ai + b−ai . O que ele conta? Conta o numero de maneira
                                                              2        2                               ´
de escolhermos duas pessoas do mesmo sexo em um grupo de ai homens e b − ai mulheres. Esse numero tamb´ m pode
                                                                                                    ´            e
                                                                     b
ser calculado contando o numero escolhas de duas pessoas quaisquer ( 2 ) retirando-se aquelas escolhas mistas (ai (b − ai )).
                          ´
Como b e impar,
         ´
                                                                                1 ≤ (b − 2ai )2
                                            1 + 4ai (b − ai ) ≤ b2
                                                                b2 − 1
                                                 ai (b − ai ) ≤
                                                                   4
Substituindo na primeira desigualdade, obtemos:
                                                      a
                                                                     b   b2 − 1                           b
                                                                       −                         ≤ k
                                                      i=0
                                                                     2      4                             2
                                             b(b − 1)    (b − 1)(b + 1)                                  b(b − 1)
                                           a          −a                                         ≤ k
                                                 2             4                                             2
                                                                  b+1
                                                            ab −                                 ≤ kb
                                                                    2
                                                                  b−1                                k
                                                                                                 ≤
                                                                   2b                                a

Exerc´cios Propostos
     ı
  1. (Olimp´ada Russa) Com os d´gitos 1 e 2 formamos 5 numeros de n d´gitos de tal forma que dois quaisquer destes
           ı                   ı                          ´              ı
     numeros coincidam em exatamente m casas decimais e n˜ o existe nenhuma casa decimal onde coincidam os 5 numeros.
      ´                                                  a                                                    ´
     Demonstre que:
                                                      2     m     3
                                                        ≤      ≤ .
                                                      5     n     5


                                                                                    5
2. Em um certo comitˆ , cada membro pertence a exatamente trˆ s subcomitˆ s e cada subcomitˆ possui exatamente trˆ s
                         e                                       e           e                  e                     e
                               ´                 ´         ´
        membros. Prove que o numero de membros e igual ao numero de subcomitˆ s.e

     3. (Olimp´ada Balcˆ nica 1997) Sejam m e n inteiros maiores que 1. Seja S um conjunto com n elementos, e sejam
                 ı            a
        A1 , A2 , . . . , Am suconjuntos de S. Assuma que para quaisquer dois elementos x e y em S, existe um conjunto Ai
        tal que ou x est´ em Ai e y n˜ o est´ em Ai ou x n˜ o est´ em Ai e y est´ em Ai . Prove que n ≤ 2m .
                            a            a    a           a      a              a


     4. (Olimp´ada Romena 2003) Um inteiro n, n ≥ 2, e bonito se existe uma fam´lia de conjuntos A1 , A2 , . . . , An de subcon-
               ı                                       ´                       ı
        juntos do conjunto {1, 2, . . . , n} tais que:

         (1) i ∈ Ai para todo i.
         (2) i ∈ Aj se e somente se j ∈ Ai , para ´ndices distintos i e j.
                                      /           ı
         (3) Ai ∩ Aj e n˜ o vazio para todos i, j ∈ {1, 2, . . . , n}.
                     ´ a

         Prove que:

         a) 7 e bonito.
              ´
         b) n e bonito se e somente se n ≥ 7.
              ´


4 Comitˆ s e Conjuntos
       e
                                                ´
O caso mais comum para usarmos contagens duplas e quando o conjunto que queremos contar possui uma estrutura de
porduto A × B, pois nesse caso:

                                                                 |(a, B)|   =         |(A, b)|
                                                           a∈A                  b∈B

        ¸˜
Nessa secao, trataremos de alguns exemplos onde esta estruturua n˜ o est´ t˜ o evidente no problema.
                                                                 a      a a

Problema 12.
1
 H´ n tipos de doce na loja do Z´ . Um dia, m amigos se juntam para comprarem k doces diferentes cada um. Cada tipo de
   a                            e
doce e comprado por r dos amigos. Cada par de tipo de doces e escolhido por t amigos. Prove que:
     ´                                                       ´

    (i) mk = nr;

    (ii) r(k − 1) = t(n − 1).

Solucao O numero de doces levados da loja e mk. Como cada doce e levado por r amigos, esse numero tamb´ m e nr.
     ¸˜     ´                                  ´                     ´                              ´           e ´
                        a                               ¸˜                      ´     ´ r
Quantos pares de doces s˜ o levados? Pela ultima informacao do enunciado, esse numero e t 2 . Por outro lado, cada amigo
                                          ´
leva k pares de doces, logo:
      2

                                                                    r             k
                                                                 t          = m
                                                                    2             2
                                                              r(r − 1)          k(k − 1)
                                                            t               = m
                                                                  2                  2
                                                              r(r − 1)        nr(k − 1)
                                                            t               =
                                                                  2                2
                                                              r(k − 1)      = t(n − 1)

Problema 13. 2 Seja X um conjunto com n elementos (n ≥ 1). Suponha que F = {A1 , A2 , . . . , Am } e uma fam´lia de
                                                                                                   ´        ı
subconjuntos de X com a propriedade que
                                                |Ai ∩ Aj | = 1,
para todos i = j. Mostre que m ≤ n.

Solucao Suponhamos inicialmente que m ≥ 2 (Quando m < 2 o problema e trivial). Al´ m disso, suponhamos que nenhum
      ¸˜                                                                           ´           e
Ai e vazio ou igual ao conjunto todo. Iremos nos concentrar nos pares (x, A) onde x ∈ A. Suponha que o elemento x pertence
   ´                                                                                       /
exatamente aos d(x) conjuntos: {A1 , A2 , . . . , Ad(x) }. Como |A ∩ Ai | = 1, para cada i ∈ {1, 2, . . . , d(x)}, deve existir um xi = x
    1 Vocˆ
         e   poder´ encontrar mais exemplos como esse estudando Block designs.
                   a
    2 Este            ´                                                        ¸˜                                                          ´
             problema e um caso particular da desigualdade de Fischer. Uma solucao elementar para este problema pode ser encontrada usando Algebra
Linear



                                                                            6
tal que xi ∈ A ∩ Ai . Al´ m disso, se i = j, xi = xj e consequentemente |A| ≥ d(x). Se m > n, m − d(x) > n − d(x) ≥ n − |A| e
                        e
portanto,
                                                          d(x)       |A|
                                                                 <         .
                                                        m − d(x)   n − |A|
                                              |A|
Para cada tal par (x, A), some o numero
                                  ´                 . Qual numeros iremos obter? Fixado A, existem n − |A| poss´veis valores
                                                            ´                                                  ı
                                            n − |A|
de x, logo:
                                                      |A|                (n − |A|)|A|
                                                            =                         =       |A|.
                                                    n − |A|                n − |A|
                                            (x,A)                   A                     A

Fixado x, existem m − d(x) possibilidade para A, portanto, a soma anterior deve ser maior que

                                                d(x)                    (m − d(x)d(x))
                                                       =                               =          d(x).
                                              m − d(x)          x
                                                                          m − d(x)            x
                                      (x,A)


Assim obtivemos um absurdo pois A |A| = x d(x) (estamos contando os elementos que foram usados nos conjuntos de
duas formas). Os casos em que algum dos Ai ’s e igual ao conjunto todo ou o conjunto vazio e deixado para o leitor.
                                              ´                                            ´

Problema 14. (Banco IMO 2004) Existem n estudantes em uma universidade, n inteiro ´mpar. Alguns estudantes se reunem
                                                                                     ı
em clubes (um estudante pode pertencer a clubes diferentes). Alguns clubes se reunem para formar algumas sociedades(um
clube pode pertencer a diferentes sociedades). Existem k sociedades. Suponham que acontecam as seguintes condicoes:
                                                                                          ¸                    ¸˜

  i) cada par de estudantes est´ em exatamente um clube.
                               a

 ii) para cada estudante e cada sociedade, o estudante est´ em exatamente um clube da sociedade,
                                                          a

iii) cada clube tem um numero ´mpar de estudantes e, al´ m disso, um clube com 2m + 1 estudantes (m > 0) est´ em
                         ´    ı                        e                                                    a
     exatamente m sociedades.

Encontre todos os valores poss´veis de k.
                              ı

Solucao Vamos contar o numero de triplas (e, C, S) onde e e um estudante, C e um clube e S e uma sociedade. Fixados
     ¸˜                     ´                                 ´                 ´               ´
e e S, existe apenas uma possibilidade para C, assim, em nossa primeira contagem obtivemos nk tais triplas. Fixado uma
                    |C| − 1
comiss˜ o C, temos
       a                    possibilidades para S, e uma vez que escolhemos C e S, temos |C| possibilidades para e(ele deve
                       2
ser um estudante em C). Assim, o numero de tais triplas e
                                     ´                    ´

                                                         |C|(|C| − 1)
                                      nk      =
                                                              2
                                                     C
                                              = total de pares de estudantes nos clubes
                                                  n
                                              =
                                                  2

                           n−1
E consequentemente, k =                                         ´        ´
                                 . Um exemplo onde este valor e atigindo e considerando apenas um clube com todos os
                             2
estudantes e k sociedades iguais formadas por esse unico clube.
                                                   ´

Exerc´cios Propostos
     ı
                                                                ˜
  1. Nove cientistas trabalham em um projeto sigiloso. Por questoes de seguranca, os planos s˜ o guardados em um cofre
                                                                               ¸             a
                                                  ´ ´
     protegido por muitos cadeados de modo que so e poss´vel abr´-los todos se houver pelo menos 5 cientistas presentes.
                                                          ı       ı
                ´    ´
       (i) Qual e o numero m´nimo poss´vel de cadeados?
                            ı         ı
       (ii) Na situacao do item (i), quantas chaves cada cientista deve ter?
                    ¸˜

  2. (Olimp´ada Russa)
           ı
       (i) Uma comiss˜ o se reuniu 40 vezes. Em cada reuni˜ o estiveram presentes 10 pessoas de tal maneira que quaisquer
                      a                                    a
           dois dos membros da comiss˜ o n˜ o estiveram juntos em mais de uma oportunidade. Demonstre que a quantidade
                                      a a
                                 a ´
           de membros da comiss˜ o e maior que 60.
       (ii) Demonstre que com 25 pessoas n˜ o se pode formar mais que 30 comissoes de 5 pessoas cada uma de modo que
                                             a                                 ˜
                                    ˜
            n˜ o existam duas comissoes que tenham mais de um membro em comum.
             a


                                                                          7
3. (OBM,1992/8) Em um torneio de xadrez cada jogador disputou uma partida com cada um dos demais participantes. A
                                                         1
      cada partida, havendo empate, cada jogador ganhou ponto; caso contr´ rio, o vencedor ganhou 1 ponto e o perdedor
                                                                            a
                                                         2
                                                                                        ´
      0 pontos. Participaram homens e mulheres e cada participante conquistou o mesmo numero de pontos contra homens
                                          ´                     ´
      que contra mulheres. Mostre que o numero de participantes e um quadrado perfeito.


5 Problemas Suplementares
                     ¸˜
Os problemas desta secao exigem mais engenhosidade.

Teorema 1. (Erdos-Ko-Rado) Seja X um conjunto com n elementos e F = {A1 , A2 , . . . , Ap } uma fam´lia de subconjuntos de
                                                                                                   ı
X que satisfazem as seguintes condicoes
                                   ¸˜
                        n
   1. |Ai | = k ≤         para todo i.
                        2
   2. Ai ∩ Aj = ∅ para todos i e j distintos.
             n−1
Ent˜ o p ≤
   a         k−1    .

Prova.Comecemos com um lema que aparentemente n˜ o apresenta nenhuma conex˜ o com o problema:
                                               a                          a

Lema. Considere um c´rculo C dividido por n pontos. Um arco de tamanho k consiste de k + 1 pontos consecutivos e k
                          ı
arestas entre eles. Seja n ≥ 2k, e suponha que existem t arcos distintos A1 , A2 , . . . , At , todos de tamanho k, tais que quaisquer
dois arcos possuem uma aresta em comum. Ent˜ o t ≤ k.
                                                 a

    A prova do lema ser´ deixada como exerc´cio para o leitor. Seja F uma fam´lia como no enunciado do teorema. Cada
                         a                     ı                              ı
permutacao c´clica dos elementos de X corresponde a uma divis˜ o em pontos e arcos, onde cada elemento do conjunto
         ¸˜ ı                                                    a
est´ entre dois pontos. Cada conjunto de k elementos pode ser visto como um arco de tamanho k formando por elementos
   a
justapostos (em uma ordem qualquer). Para cada tal permutacao, podemos calcular quantos arcos de tamanho k correspon-
                                                            ¸˜
dem a conjuntos da fam´lia. Pelo lema, esse numero e menor ou igual a k. Como existem (n − 1)! permutacoes, a quantidade
                        ı                    ´     ´                 `                                ¸˜
m´ xima de conjuntos que podem ser encontrados nessas permutacoes e k(n − 1)!. Certamente nessa contagem, alguns con-
  a                                                             ¸˜ ´
                                     ´
juntos foram contados v´ rias vezes. E agora que usaremos uma contagem dupla. Fixado um conjunto A, seus elementos
                          a
podem estar arranjandos para formarem um arco de tamanho k de k! maneiras. Os outros elementos do conjunto podem ser
permutados nas posicoes que restaram de (n − k)! maneiras. Ou seja, cada conjunto e contado exatamente k!(n − k)! vezes.
                     ¸˜                                                            ´
Como existem p conjuntos, temos:

                                                     pk!(n − k)!        ≤ k(n − 1)!;
                                                                           n−1
                                                                   p    ≤
                                                                            k−1

Problema 15. (Olimp´ada do Leningrado 1987) Sejam A1 , A2 , . . . , As uma fam´lia de subconjuntos de {1, 2, . . . , n} tais que
                       ı                                                      ı
|Ai | = ai . Sabemos que nenhum desses subconjuntos cont´ m outro subconjunto da fam´lia. Prove que
                                                        e                              ı
                                                     −1            −1                  −1
                                                n             n                   n
                                                          +             + ... +             ≤1
                                                a1            a2                  as



                                        ¸˜
Problema 16. (IMO 2005) Numa competicao de matem´ tica na qual foram propostos 6 problemas, quaisquer dois prob-
                                                       a
lemas foram resolvidos por mais de 2/5 dos estudantes. Al´ m disso, nenhum estudante resolveu todos os 6 problemas.
                                                          e
Mostre que existem pelo menos 2 estudantes que resolveram 5 problemas cada um.


Problema 17. Alguns asteriscos est˜ o escritos nas casas de um tabuleiro m × n(m < n), de modo que existe pelo menos
                                      a
um asterisco em cada coluna. Mostre que que existe um asterisco A tal que lA > cA , onde la e cA denotam as quantidade de
asteriscos na linha e coluna de A, respectivamente.


Problema 18. (Olimp´ada Iberoamericana 2001) Sejam S um conjunto de n elementos e S1 , S2 , . . . , Sk subconjuntos de
                       ı
S(k ≥ 2), tais que cada um deles tˆ m pelo menos r elementos. Demonstre que existem i e j, com i = j, tais que a quantidade
                                  e
                                                                nk
de elementos em comum entre Si e Sj e maior ou igual a r −
                                       ´               `              .
                                                             4(k − 1)


                                                                        8
Problema 19. (IMO 1989) Sejam n e k dois inteiros positivos e seja S um conjunto de n pontos num plano tais que

   1. n˜ o haja trˆ s pontos de S que sejam colineares;
       a          e

  2. para qualquer ponto P de S, h´ pelo menos k pontos de S que s˜ o equidistantes de P .
                                  a                               a
                    √
Prove que k < 1/2 + 2n.


Problema 20. (IMO 1987) 3 Seja pn (k) o total de permutacoes do conjunto {1, 2, 3, . . . , n} que contˆ m exatamente k pontos
                                                        ¸˜                                            e
fixos. Prove que
                                                               n
                                                                    k · pn (k) = n!.
                                                              k=1

Observacao: uma permutacao f de um conjunto S e uma funcao bijetora de S em S; um elemento i de S e chamado ponto
        ¸˜                ¸˜                  ´        ¸˜                                         ´
fixo da permutacao f se f (i) = i.
              ¸˜




   3 O enunciado mais geral desse problema e o seguinte: Seja G um grupo finito agindo sobre um conjuno M . Sejam k o numero de orbitas em que M e
                                              ´                                                                       ´        ´                ´
dividido por esta acao e F ix(g), g ∈ G, o numero de pontos fixos de g. Ent˜ o:
                   ¸˜                       ´                             a

                                                                    F ix(g) = k · |G|.
                                                              g∈G

     ´
Esse e o lema de Burnside.


                                                                         9

Contenu connexe

En vedette

Slide eproinfo 3.4
Slide eproinfo 3.4Slide eproinfo 3.4
Slide eproinfo 3.4TEKABARBOSA
 
3.tenseในบทความภาษาอังกฤษธุรกิจ
3.tenseในบทความภาษาอังกฤษธุรกิจ3.tenseในบทความภาษาอังกฤษธุรกิจ
3.tenseในบทความภาษาอังกฤษธุรกิจrajamongala rattanakosin wangklaikangwon
 
Materias primas
Materias primasMaterias primas
Materias primashenar5
 
Translation by omission
Translation by omissionTranslation by omission
Translation by omissionKhanhHoa Tran
 
Introdução a Redes de Computadores - 2 - Nível de aplicação (HTTP)
Introdução a Redes de Computadores - 2 - Nível de aplicação (HTTP)Introdução a Redes de Computadores - 2 - Nível de aplicação (HTTP)
Introdução a Redes de Computadores - 2 - Nível de aplicação (HTTP)Andre Peres
 
Apresentação projeto africa
Apresentação projeto africaApresentação projeto africa
Apresentação projeto africamachadogui
 
Karol trabajo de sistemas
Karol trabajo de sistemasKarol trabajo de sistemas
Karol trabajo de sistemaskarol ruiz
 
Afiliação de profissionais
Afiliação de profissionaisAfiliação de profissionais
Afiliação de profissionaisanonymoushahaha
 
Viatge final de curs
Viatge final de cursViatge final de curs
Viatge final de cursmiquelvissa
 

En vedette (18)

Slide eproinfo 3.4
Slide eproinfo 3.4Slide eproinfo 3.4
Slide eproinfo 3.4
 
Rodrigo schardong
Rodrigo schardongRodrigo schardong
Rodrigo schardong
 
Bondia Lleida 06062012
Bondia Lleida 06062012Bondia Lleida 06062012
Bondia Lleida 06062012
 
Bondia Lleida 21102011
Bondia Lleida 21102011Bondia Lleida 21102011
Bondia Lleida 21102011
 
3.tenseในบทความภาษาอังกฤษธุรกิจ
3.tenseในบทความภาษาอังกฤษธุรกิจ3.tenseในบทความภาษาอังกฤษธุรกิจ
3.tenseในบทความภาษาอังกฤษธุรกิจ
 
costadorada
costadoradacostadorada
costadorada
 
Materias primas
Materias primasMaterias primas
Materias primas
 
Translation by omission
Translation by omissionTranslation by omission
Translation by omission
 
charlin chaplin
charlin chaplincharlin chaplin
charlin chaplin
 
Fotos
FotosFotos
Fotos
 
Introdução a Redes de Computadores - 2 - Nível de aplicação (HTTP)
Introdução a Redes de Computadores - 2 - Nível de aplicação (HTTP)Introdução a Redes de Computadores - 2 - Nível de aplicação (HTTP)
Introdução a Redes de Computadores - 2 - Nível de aplicação (HTTP)
 
Apresentação projeto africa
Apresentação projeto africaApresentação projeto africa
Apresentação projeto africa
 
As Sombras chinesas
As Sombras chinesasAs Sombras chinesas
As Sombras chinesas
 
Lista 3
Lista 3 Lista 3
Lista 3
 
Karol trabajo de sistemas
Karol trabajo de sistemasKarol trabajo de sistemas
Karol trabajo de sistemas
 
Bondia.cat 28/02/2013
Bondia.cat 28/02/2013Bondia.cat 28/02/2013
Bondia.cat 28/02/2013
 
Afiliação de profissionais
Afiliação de profissionaisAfiliação de profissionais
Afiliação de profissionais
 
Viatge final de curs
Viatge final de cursViatge final de curs
Viatge final de curs
 

Similaire à Contagensduplas

EQUAÇÃO DE 1º GRAU
EQUAÇÃO DE 1º GRAUEQUAÇÃO DE 1º GRAU
EQUAÇÃO DE 1º GRAUguest941f1e3
 
Implementação modulo3
Implementação modulo3Implementação modulo3
Implementação modulo3inechidias
 
Fracções próprias,impróprias,decimais e equivalentes
Fracções próprias,impróprias,decimais e equivalentesFracções próprias,impróprias,decimais e equivalentes
Fracções próprias,impróprias,decimais e equivalentesmariacferreira
 
Frações desenvolvimento
Frações desenvolvimentoFrações desenvolvimento
Frações desenvolvimentomarcommendes
 
Geometriamar - Prof. Marcelo Lopes - Permutação e Combinação
Geometriamar - Prof. Marcelo Lopes - Permutação e CombinaçãoGeometriamar - Prof. Marcelo Lopes - Permutação e Combinação
Geometriamar - Prof. Marcelo Lopes - Permutação e Combinaçãogeometriamar.com.br
 
Implementação modulo3
Implementação modulo3Implementação modulo3
Implementação modulo3inechidias
 
Implementação modulo3
Implementação modulo3Implementação modulo3
Implementação modulo3inechidias
 
Implementação modulo3
Implementação modulo3Implementação modulo3
Implementação modulo3inechidias
 
Implementação modulo3
Implementação modulo3Implementação modulo3
Implementação modulo3inechidias
 
Implementação modulo3
Implementação modulo3Implementação modulo3
Implementação modulo3inechidias
 
Implementação modulo3
Implementação modulo3Implementação modulo3
Implementação modulo3inechidias
 
Implementação modulo3
Implementação modulo3Implementação modulo3
Implementação modulo3inechidias
 
Implementação modulo3
Implementação modulo3Implementação modulo3
Implementação modulo3inechidias
 
Bndes racioc logico_exe_guilherme_neves_aula 00
Bndes racioc logico_exe_guilherme_neves_aula 00Bndes racioc logico_exe_guilherme_neves_aula 00
Bndes racioc logico_exe_guilherme_neves_aula 00Paulo Roberto
 
Formulas para numeros primos 1ed - Eric Campos Bastos Guedes
Formulas para numeros primos 1ed - Eric Campos Bastos GuedesFormulas para numeros primos 1ed - Eric Campos Bastos Guedes
Formulas para numeros primos 1ed - Eric Campos Bastos Guedesericnalanhouse2
 
Formação matemática junho
Formação matemática junhoFormação matemática junho
Formação matemática junhoDyone Andrade
 
Problemas com problemas com equações
Problemas com problemas com equaçõesProblemas com problemas com equações
Problemas com problemas com equaçõesRita Sousa
 

Similaire à Contagensduplas (20)

EQUAÇÃO DE 1º GRAU
EQUAÇÃO DE 1º GRAUEQUAÇÃO DE 1º GRAU
EQUAÇÃO DE 1º GRAU
 
Equacao 1 Grau
Equacao 1 GrauEquacao 1 Grau
Equacao 1 Grau
 
Implementação modulo3
Implementação modulo3Implementação modulo3
Implementação modulo3
 
Fracções próprias,impróprias,decimais e equivalentes
Fracções próprias,impróprias,decimais e equivalentesFracções próprias,impróprias,decimais e equivalentes
Fracções próprias,impróprias,decimais e equivalentes
 
Frações desenvolvimento
Frações desenvolvimentoFrações desenvolvimento
Frações desenvolvimento
 
Geometriamar - Prof. Marcelo Lopes - Permutação e Combinação
Geometriamar - Prof. Marcelo Lopes - Permutação e CombinaçãoGeometriamar - Prof. Marcelo Lopes - Permutação e Combinação
Geometriamar - Prof. Marcelo Lopes - Permutação e Combinação
 
Implementação modulo3
Implementação modulo3Implementação modulo3
Implementação modulo3
 
Implementação modulo3
Implementação modulo3Implementação modulo3
Implementação modulo3
 
Implementação modulo3
Implementação modulo3Implementação modulo3
Implementação modulo3
 
Implementação modulo3
Implementação modulo3Implementação modulo3
Implementação modulo3
 
Implementação modulo3
Implementação modulo3Implementação modulo3
Implementação modulo3
 
Implementação modulo3
Implementação modulo3Implementação modulo3
Implementação modulo3
 
Implementação modulo3
Implementação modulo3Implementação modulo3
Implementação modulo3
 
Implementação modulo3
Implementação modulo3Implementação modulo3
Implementação modulo3
 
8 simulado 2021
8 simulado   20218 simulado   2021
8 simulado 2021
 
Bndes racioc logico_exe_guilherme_neves_aula 00
Bndes racioc logico_exe_guilherme_neves_aula 00Bndes racioc logico_exe_guilherme_neves_aula 00
Bndes racioc logico_exe_guilherme_neves_aula 00
 
Formulas para numeros primos 1ed - Eric Campos Bastos Guedes
Formulas para numeros primos 1ed - Eric Campos Bastos GuedesFormulas para numeros primos 1ed - Eric Campos Bastos Guedes
Formulas para numeros primos 1ed - Eric Campos Bastos Guedes
 
Conjunto dos números naturais
Conjunto dos números naturaisConjunto dos números naturais
Conjunto dos números naturais
 
Formação matemática junho
Formação matemática junhoFormação matemática junho
Formação matemática junho
 
Problemas com problemas com equações
Problemas com problemas com equaçõesProblemas com problemas com equações
Problemas com problemas com equações
 

Plus de Dayanne Sousa

Matemática e física
Matemática e físicaMatemática e física
Matemática e físicaDayanne Sousa
 
.Problemas de eletricidade.
.Problemas de eletricidade..Problemas de eletricidade.
.Problemas de eletricidade.Dayanne Sousa
 
Exercícios sobre função
Exercícios sobre funçãoExercícios sobre função
Exercícios sobre funçãoDayanne Sousa
 
Professor helanderson }
Professor helanderson }Professor helanderson }
Professor helanderson }Dayanne Sousa
 
Professor helanderson sousa...
Professor helanderson sousa...Professor helanderson sousa...
Professor helanderson sousa...Dayanne Sousa
 
Bad girls vs Bad boysx
Bad girls vs Bad boysxBad girls vs Bad boysx
Bad girls vs Bad boysxDayanne Sousa
 
Gases perfeitos e Termodinâmicax
Gases perfeitos e TermodinâmicaxGases perfeitos e Termodinâmicax
Gases perfeitos e TermodinâmicaxDayanne Sousa
 
PROFESSOR HELANDERSON SOUSAx
PROFESSOR HELANDERSON SOUSAxPROFESSOR HELANDERSON SOUSAx
PROFESSOR HELANDERSON SOUSAxDayanne Sousa
 
PROFESSOR HELANDERSON SOUSA.pdf
PROFESSOR HELANDERSON SOUSA.pdfPROFESSOR HELANDERSON SOUSA.pdf
PROFESSOR HELANDERSON SOUSA.pdfDayanne Sousa
 
Gases perfeitos e termodinâmica
Gases perfeitos e termodinâmicaGases perfeitos e termodinâmica
Gases perfeitos e termodinâmicaDayanne Sousa
 
Gases perfeitos e termodinâmica
Gases perfeitos e termodinâmicaGases perfeitos e termodinâmica
Gases perfeitos e termodinâmicaDayanne Sousa
 
Gases perfeitos e termodinâmica
Gases perfeitos e termodinâmicaGases perfeitos e termodinâmica
Gases perfeitos e termodinâmicaDayanne Sousa
 
Gases perfeitos e termodinamica
Gases perfeitos e termodinamicaGases perfeitos e termodinamica
Gases perfeitos e termodinamicaDayanne Sousa
 

Plus de Dayanne Sousa (20)

Matemática e física
Matemática e físicaMatemática e física
Matemática e física
 
Simulado enem
Simulado enemSimulado enem
Simulado enem
 
Exercícios
ExercíciosExercícios
Exercícios
 
.Problemas de eletricidade.
.Problemas de eletricidade..Problemas de eletricidade.
.Problemas de eletricidade.
 
Exercícios sobre função
Exercícios sobre funçãoExercícios sobre função
Exercícios sobre função
 
Professor helanderson }
Professor helanderson }Professor helanderson }
Professor helanderson }
 
Professor helanderson sousa...
Professor helanderson sousa...Professor helanderson sousa...
Professor helanderson sousa...
 
óPtica
óPticaóPtica
óPtica
 
Gravitação
GravitaçãoGravitação
Gravitação
 
Bad girls vs Bad boysx
Bad girls vs Bad boysxBad girls vs Bad boysx
Bad girls vs Bad boysx
 
lista3
lista3lista3
lista3
 
Gases perfeitos e Termodinâmicax
Gases perfeitos e TermodinâmicaxGases perfeitos e Termodinâmicax
Gases perfeitos e Termodinâmicax
 
Untitled document
Untitled documentUntitled document
Untitled document
 
PROFESSOR HELANDERSON SOUSAx
PROFESSOR HELANDERSON SOUSAxPROFESSOR HELANDERSON SOUSAx
PROFESSOR HELANDERSON SOUSAx
 
PROFESSOR HELANDERSON SOUSA.pdf
PROFESSOR HELANDERSON SOUSA.pdfPROFESSOR HELANDERSON SOUSA.pdf
PROFESSOR HELANDERSON SOUSA.pdf
 
Gases perfeitos e termodinâmica
Gases perfeitos e termodinâmicaGases perfeitos e termodinâmica
Gases perfeitos e termodinâmica
 
Gravitação
GravitaçãoGravitação
Gravitação
 
Gases perfeitos e termodinâmica
Gases perfeitos e termodinâmicaGases perfeitos e termodinâmica
Gases perfeitos e termodinâmica
 
Gases perfeitos e termodinâmica
Gases perfeitos e termodinâmicaGases perfeitos e termodinâmica
Gases perfeitos e termodinâmica
 
Gases perfeitos e termodinamica
Gases perfeitos e termodinamicaGases perfeitos e termodinamica
Gases perfeitos e termodinamica
 

Dernier

Gerenciando a Aprendizagem Organizacional
Gerenciando a Aprendizagem OrganizacionalGerenciando a Aprendizagem Organizacional
Gerenciando a Aprendizagem OrganizacionalJacqueline Cerqueira
 
Prática de interpretação de imagens de satélite no QGIS
Prática de interpretação de imagens de satélite no QGISPrática de interpretação de imagens de satélite no QGIS
Prática de interpretação de imagens de satélite no QGISVitor Vieira Vasconcelos
 
A galinha ruiva sequencia didatica 3 ano
A  galinha ruiva sequencia didatica 3 anoA  galinha ruiva sequencia didatica 3 ano
A galinha ruiva sequencia didatica 3 anoandrealeitetorres
 
QUARTA - 1EM SOCIOLOGIA - Aprender a pesquisar.pptx
QUARTA - 1EM SOCIOLOGIA - Aprender a pesquisar.pptxQUARTA - 1EM SOCIOLOGIA - Aprender a pesquisar.pptx
QUARTA - 1EM SOCIOLOGIA - Aprender a pesquisar.pptxIsabellaGomes58
 
Modelos de Desenvolvimento Motor - Gallahue, Newell e Tani
Modelos de Desenvolvimento Motor - Gallahue, Newell e TaniModelos de Desenvolvimento Motor - Gallahue, Newell e Tani
Modelos de Desenvolvimento Motor - Gallahue, Newell e TaniCassio Meira Jr.
 
ALMANANHE DE BRINCADEIRAS - 500 atividades escolares
ALMANANHE DE BRINCADEIRAS - 500 atividades escolaresALMANANHE DE BRINCADEIRAS - 500 atividades escolares
ALMANANHE DE BRINCADEIRAS - 500 atividades escolaresLilianPiola
 
Cenários de Aprendizagem - Estratégia para implementação de práticas pedagógicas
Cenários de Aprendizagem - Estratégia para implementação de práticas pedagógicasCenários de Aprendizagem - Estratégia para implementação de práticas pedagógicas
Cenários de Aprendizagem - Estratégia para implementação de práticas pedagógicasRosalina Simão Nunes
 
A experiência amorosa e a reflexão sobre o Amor.pptx
A experiência amorosa e a reflexão sobre o Amor.pptxA experiência amorosa e a reflexão sobre o Amor.pptx
A experiência amorosa e a reflexão sobre o Amor.pptxfabiolalopesmartins1
 
Currículo escolar na perspectiva da educação inclusiva.pdf
Currículo escolar na perspectiva da educação inclusiva.pdfCurrículo escolar na perspectiva da educação inclusiva.pdf
Currículo escolar na perspectiva da educação inclusiva.pdfIedaGoethe
 
Cultura e Sociedade - Texto de Apoio.pdf
Cultura e Sociedade - Texto de Apoio.pdfCultura e Sociedade - Texto de Apoio.pdf
Cultura e Sociedade - Texto de Apoio.pdfaulasgege
 
Simulado 1 Etapa - 2024 Proximo Passo.pdf
Simulado 1 Etapa - 2024 Proximo Passo.pdfSimulado 1 Etapa - 2024 Proximo Passo.pdf
Simulado 1 Etapa - 2024 Proximo Passo.pdfEditoraEnovus
 
ATIVIDADE AVALIATIVA VOZES VERBAIS 7º ano.pptx
ATIVIDADE AVALIATIVA VOZES VERBAIS 7º ano.pptxATIVIDADE AVALIATIVA VOZES VERBAIS 7º ano.pptx
ATIVIDADE AVALIATIVA VOZES VERBAIS 7º ano.pptxOsnilReis1
 
Educação São Paulo centro de mídias da SP
Educação São Paulo centro de mídias da SPEducação São Paulo centro de mídias da SP
Educação São Paulo centro de mídias da SPanandatss1
 
PPT _ Módulo 3_Direito Comercial_2023_2024.pdf
PPT _ Módulo 3_Direito Comercial_2023_2024.pdfPPT _ Módulo 3_Direito Comercial_2023_2024.pdf
PPT _ Módulo 3_Direito Comercial_2023_2024.pdfAnaGonalves804156
 
Atividade com a letra da música Meu Abrigo
Atividade com a letra da música Meu AbrigoAtividade com a letra da música Meu Abrigo
Atividade com a letra da música Meu AbrigoMary Alvarenga
 
Habilidades Motoras Básicas e Específicas
Habilidades Motoras Básicas e EspecíficasHabilidades Motoras Básicas e Específicas
Habilidades Motoras Básicas e EspecíficasCassio Meira Jr.
 
Sociologia Contemporânea - Uma Abordagem dos principais autores
Sociologia Contemporânea - Uma Abordagem dos principais autoresSociologia Contemporânea - Uma Abordagem dos principais autores
Sociologia Contemporânea - Uma Abordagem dos principais autoresaulasgege
 
geografia 7 ano - relevo, altitude, topos do mundo
geografia 7 ano - relevo, altitude, topos do mundogeografia 7 ano - relevo, altitude, topos do mundo
geografia 7 ano - relevo, altitude, topos do mundonialb
 
trabalho wanda rocha ditadura
trabalho wanda rocha ditaduratrabalho wanda rocha ditadura
trabalho wanda rocha ditaduraAdryan Luiz
 

Dernier (20)

Gerenciando a Aprendizagem Organizacional
Gerenciando a Aprendizagem OrganizacionalGerenciando a Aprendizagem Organizacional
Gerenciando a Aprendizagem Organizacional
 
Prática de interpretação de imagens de satélite no QGIS
Prática de interpretação de imagens de satélite no QGISPrática de interpretação de imagens de satélite no QGIS
Prática de interpretação de imagens de satélite no QGIS
 
A galinha ruiva sequencia didatica 3 ano
A  galinha ruiva sequencia didatica 3 anoA  galinha ruiva sequencia didatica 3 ano
A galinha ruiva sequencia didatica 3 ano
 
QUARTA - 1EM SOCIOLOGIA - Aprender a pesquisar.pptx
QUARTA - 1EM SOCIOLOGIA - Aprender a pesquisar.pptxQUARTA - 1EM SOCIOLOGIA - Aprender a pesquisar.pptx
QUARTA - 1EM SOCIOLOGIA - Aprender a pesquisar.pptx
 
Modelos de Desenvolvimento Motor - Gallahue, Newell e Tani
Modelos de Desenvolvimento Motor - Gallahue, Newell e TaniModelos de Desenvolvimento Motor - Gallahue, Newell e Tani
Modelos de Desenvolvimento Motor - Gallahue, Newell e Tani
 
ALMANANHE DE BRINCADEIRAS - 500 atividades escolares
ALMANANHE DE BRINCADEIRAS - 500 atividades escolaresALMANANHE DE BRINCADEIRAS - 500 atividades escolares
ALMANANHE DE BRINCADEIRAS - 500 atividades escolares
 
Cenários de Aprendizagem - Estratégia para implementação de práticas pedagógicas
Cenários de Aprendizagem - Estratégia para implementação de práticas pedagógicasCenários de Aprendizagem - Estratégia para implementação de práticas pedagógicas
Cenários de Aprendizagem - Estratégia para implementação de práticas pedagógicas
 
A experiência amorosa e a reflexão sobre o Amor.pptx
A experiência amorosa e a reflexão sobre o Amor.pptxA experiência amorosa e a reflexão sobre o Amor.pptx
A experiência amorosa e a reflexão sobre o Amor.pptx
 
Currículo escolar na perspectiva da educação inclusiva.pdf
Currículo escolar na perspectiva da educação inclusiva.pdfCurrículo escolar na perspectiva da educação inclusiva.pdf
Currículo escolar na perspectiva da educação inclusiva.pdf
 
Cultura e Sociedade - Texto de Apoio.pdf
Cultura e Sociedade - Texto de Apoio.pdfCultura e Sociedade - Texto de Apoio.pdf
Cultura e Sociedade - Texto de Apoio.pdf
 
Simulado 1 Etapa - 2024 Proximo Passo.pdf
Simulado 1 Etapa - 2024 Proximo Passo.pdfSimulado 1 Etapa - 2024 Proximo Passo.pdf
Simulado 1 Etapa - 2024 Proximo Passo.pdf
 
XI OLIMPÍADAS DA LÍNGUA PORTUGUESA -
XI OLIMPÍADAS DA LÍNGUA PORTUGUESA      -XI OLIMPÍADAS DA LÍNGUA PORTUGUESA      -
XI OLIMPÍADAS DA LÍNGUA PORTUGUESA -
 
ATIVIDADE AVALIATIVA VOZES VERBAIS 7º ano.pptx
ATIVIDADE AVALIATIVA VOZES VERBAIS 7º ano.pptxATIVIDADE AVALIATIVA VOZES VERBAIS 7º ano.pptx
ATIVIDADE AVALIATIVA VOZES VERBAIS 7º ano.pptx
 
Educação São Paulo centro de mídias da SP
Educação São Paulo centro de mídias da SPEducação São Paulo centro de mídias da SP
Educação São Paulo centro de mídias da SP
 
PPT _ Módulo 3_Direito Comercial_2023_2024.pdf
PPT _ Módulo 3_Direito Comercial_2023_2024.pdfPPT _ Módulo 3_Direito Comercial_2023_2024.pdf
PPT _ Módulo 3_Direito Comercial_2023_2024.pdf
 
Atividade com a letra da música Meu Abrigo
Atividade com a letra da música Meu AbrigoAtividade com a letra da música Meu Abrigo
Atividade com a letra da música Meu Abrigo
 
Habilidades Motoras Básicas e Específicas
Habilidades Motoras Básicas e EspecíficasHabilidades Motoras Básicas e Específicas
Habilidades Motoras Básicas e Específicas
 
Sociologia Contemporânea - Uma Abordagem dos principais autores
Sociologia Contemporânea - Uma Abordagem dos principais autoresSociologia Contemporânea - Uma Abordagem dos principais autores
Sociologia Contemporânea - Uma Abordagem dos principais autores
 
geografia 7 ano - relevo, altitude, topos do mundo
geografia 7 ano - relevo, altitude, topos do mundogeografia 7 ano - relevo, altitude, topos do mundo
geografia 7 ano - relevo, altitude, topos do mundo
 
trabalho wanda rocha ditadura
trabalho wanda rocha ditaduratrabalho wanda rocha ditadura
trabalho wanda rocha ditadura
 

Contagensduplas

  • 1. C ONTAGENS D UPLAS S AMUEL F EITOSA ´ ´ Em um grande numero de problemas de combinatoria precisamos contar algum conjunto de pelo menos duas maneiras ´ diferentes. Nosso proposito ser´ estudar algumas formas de fazer tais contagens. O leitor deve PENSAR bastante em cada a ¸˜ ¸˜ ´ problema antes de ver a solucao. As secoes est˜ o organizadas em ordem crescente de dificuldade, sendo a ultima bem mais a avancada que as demais. ¸ 1 Grafos a ´ Problema 1. Em Brasilˆ ndia existem apenas 9 casas muito distantes entre si. E poss´vel que cada casa esteja ligada a exata- ı mente 7 outras casas atrav´ s de estradas? e Solucao N˜ o e poss´vel. Some a quantidade de estradas que saem de cada casa. Bem, facilmente obtemos 7 × 9 = 63 ¸˜ a ´ ı ´ estradas. Como cada estrada liga duas cidades, a contagem que fizemos contou cada estrada duas vezes. Logo o numero obtido teria que ser par. a ´ Ser´ que cada casa estar ligada a exatamente 7 outras foi crucial? E poss´vel revolvermos o problema anterior com um ı enunciado mais geral: Problema 2. Prove que numa festa com n pessoas, o numero de pessoas que conhecem um numero ´mpar de outras pessoas ´ ´ ı ´ na festa e par. Solucao Numere as pessoas de 1 at´ n e denote por di o numero de amigos da pessoa i. Imagine que existe um fio entre ¸˜ e ´ duas pessoas que se conhecem. Se E denota a quantidade de fios, temos d1 + d2 + . . . + dn = 2E, ´ ´ pois cada fio e contado duas vezes, um para cada ponta. Como o lado direito e par, no lado esquerdo devemos ter uma quantidade par de numeros ´mpares. ´ ı ´ Para professores. E recomend´ vel que, na sala de aula, o professor tente primeiro introduzir as ideias envolvidas na con- a ¸˜ ´ tagem dupla anterior sem introduzir notacao desnecess´ ria, por exemplo, ao explicar o que e um grafo antes mesmo de a resolver alguns problemas. Enunciados envolvendo objetos completamente diferentes, pessoas e casas como nos exemplos ´ anteriores, podem ser uteis para levar o aluno a ituir que as ideias s˜ o mais gerais e abstratas. a Problema 3. Prove que numa festa com 2n, pessoas existem duas com um numero par de amigos em comum. ´ Solucao Suponha que quaisquer duas pessoas tenham um numero ´mpar de amigos em comum e seja A um dos partici- ¸˜ ´ ı pantes da festa. Seja M = {F1 , F2 , . . . , Fk } o conjunto dos amigos de A. Considere uma nova festa restrita apenas ao conjunto M . Como cada Fi tem um numero ´mpar de amigos em comum com A, na nova festa, cada Fi possui um numero ´mpar de ´ ı ´ ı amigos. Pelo exemplo anterior, k deve ser par. O mesmo argumento vale para qualquer pessoa na festa e consequentemente todos conhecem um numero par de pessoas. Peca para cada um dos amigos de A fazerem uma lista de seus amigos difer- ´ ¸ entes de A. A soma da quantidade de nomes listados e par, pois e uma soma de uma quantidade par (igual a k) de numeros ´ ´ ´ ´mpares (cada Fi possui um numero ´mpar de amigos diferentes de A). Agora comparemos o numero de aparicoes de cada ı ´ ı ´ ¸˜ uma das 2n−1 pessoas diferentes de A nessas listas. Se cada uma delas aparecer em um numero ´mpar de listas, a soma total ´ ı de todos os nomes em todas as listas seria ´mpar (Lembre-se que a soma de uma quantidade ´mpar de numeros ´mpares e ı ı ´ ı ´ ı ´ ¸˜ ´ ´mpar!). Mas isso e uma contradicao. Logo, existem duas pessoas na festa com um numero par de amigos em comum. a ´ Problema 4. (Lema de Sperner) Um triˆ ngulo e dividido em triˆ ngulos menores de modo que quaisquer dois dentre os a triˆ ngulos menores ou n˜ o tˆ m ponto em comum, ou tˆ m um v´ rtice em comum, ou tˆ m um lado (completo) em comum. a a e e e e Os v´ rtices do triˆ ngulo maior s˜ o numerados: 1, 2, 3. Os v´ rtices dos triˆ ngulos menores tamb´ m s˜ o numerados: 1, 2 ou 3. e a a e a e a A numeracao e arbitr´ ria, exceto que os v´ rtices sobre o lado do triˆ ngulo maior oposto ao v´ rtice i (i = 1, 2, 3) n˜ o podem ¸˜ ´ a e a e a 1
  • 2. receber o numero i (veja a figura). Mostre que entre os triˆ ngulos menores existe um cujos v´ rtices s˜ o numerados 1, 2, 3. ´ a e a (Colocar Figura) ¸˜ Solucao Uma boa estrat´ gia seria pensar numa vers˜ o particular do problema. Olhando para o bordo do triˆ ngulo, temos e a a ¸˜ uma situacao semelhante ao problema inicial com uma dimens˜ o e uma cor a menos. Consideremos, por exemplo, o lado a dos v´ rtices 1 e 2, poder´amos provar que dentre os segmentos da divis˜ o deste lado, sempre existe um numero ´mpar de e ı a ´ ı segmentos com as cores 1 e 2. Imagine uma pessoa com uma bandeira abaixada no v´ rtice 1 caminhando em direcao ao e ¸˜ v´ rtice 2. Ao passar por um v´ rtice vermelho, a pessoa deve levantar a bandeira e ao passar por um v´ rtice branco a pessoa e e e deve abaixar a bandeira. Ao final do trajeto, a bandeira estar´ abaixada e consequentemente a pessoa ter´ realizado um a a numero ´mpar de movimentos de abaixar e levantar a bandeira. Mas cada movimento de abaixar ou levantar a bandeira ´ ı corresponde a um segmento com as cores 1 e 2. Outro modo de ver isso, e perceber que a adicao de um vertice de qualquer ´ ¸˜ uma das duas cores, n˜ o altera a paridade da quantidade de segmentos com v´ rtices de cores diferentes. Agora tentemos a e usar essa informacao para resolver o problema. Contaremos o numero de segmentos 12 (com algumas repeticoes). Eles ¸˜ ´ ¸˜ aparecem nos triˆ ngulos de v´ rtices 123, 122 e 112. Digamos que h´ x triˆ ngulos 123, y triˆ ngulos 122 e z triˆ ngulos 112. a e a a a a Observe que os segmentos internos ao triˆ ngulo grande s˜ o contados duas vezes (eles s˜ o comuns a dois triˆ ngulos) e os a a a a segmentos do lado do triˆ ngulo grande, somente uma vez. Notemos tamb´ m que os segmentos 12 aparecem duas vezes nos a e triˆ ngulos 122 e 112 e uma vez nos triˆ ngulos 123. Assim, a a 2 × segmentos interiores 12 + segmentos nos lados 12 = numero de segmentos 12 = x + 2y + 2z. ´ Como existe uma quantidade ´mpar de segmentos nos lados, conclu´mos que x e ´mpar. ı ı ´ı Exerc´cios Propostos ı 1. Em uma festa com 23 pessoas, e poss´vel que cada um possua 1, 3 ou 5 amigos na festa? ´ ı ´ 2. E poss´vel desenhar 9 segmentos de reta no plano de tal forma que cada um intersecta exatamente 3 outros? ı a a ´ 3. Existem 1000 cidades em Brazilˆ ndia e alguns pares de cidades s˜ o ligadas por uma estrada de terra. E poss´vel viajar ı de uma cidade para qualquer outra atrav´ s das estradas de terra. Prove que o governo de Brazilˆ ndia pode pavimentar e a algumas estradas de modo que de cada cidade saia um numero ´mpar de estradas pavimentadas. ´ ı 2 Identidades Binomiais ´ ´ ´ Uma excelente maneira de valorizar o significado de um numero binomial e, logo apos defin´-lo, estudar propriedades deste ı a ¸ ´ objeto sem necessariamente calcul´ -lo explicitamente. Facamos o mesmo aqui: O numero de maneiras de selecionarmos um subconjunto de r elementos distintos de um conjunto de n elementos distintos, onde a ordem da selecao n˜ o importa, e ¸˜ a ´ denotado por n . r ¸˜ Problema 5. Prove as seguintes afirmacoes: n n n+1 ¸˜ 1. (Relacao de Stifel) + = . r r+1 r+1 2n + 2 2n 2n 2n 2. = +2 + . n+1 n+1 n n−1 ¸˜ Solucao 1. O lado direito conta o numero de maneiras de escolhermos r + 1 pessoas em um grupo de n + 1. Podemos dividir essas ´ escolhas em dois grupos: aquelas que cont´ m um certo indiv´duo ( n ) e aquelas que n˜ o o cont´ m ( r+1 ). e ı r a e n 2. O lado esquerdo conta o numero de maneiras de escolhermos n + 1 pessoas no grupo da 2n + 2 pessoas que est˜ o ´ a comemorando o aniver´ rio da Ana e do jo˜ o. Essas escolhas se dividem em trˆ s grupos, aquelas que n˜ o cont´ m Ana a a e a e e Jo˜ o ( n+1 ), aquelas que cont´ m apenas um dos dois (2 2n ) e aquelas que cont´ m ambos ( n−1 ). a 2n e n e 2n Problema 6. (Teorema das Colunas) Mostre que: k n+i n+k+1 = . i=0 n n+1 2
  • 3. Solucao Suponha que exista uma fila de n + 1 + k pessoas. As primeiras k + 1 pessoas s˜ o c0 , c1 , . . . , ck e est˜ o ordenadas ¸˜ a a pelas suas alturas. O lado direito conta o numero de maneiras de escolhermos n + 1 pessoas nesse grupo. Certamente ´ precisaremos escolher algu´ m do conjunto {c0 , c1 , . . . , ck } = C. Seja Ci o conjunto das escolhas em que o menor elemento de e C escolhido e ci . Qualquer escolha faz parte de algum desses Ci ’s. Para calcular o numero de elementos de Ci , veja que ci ´ ´ deve fazer parte dessa escolha e os outros n elementos devem ser escolhidos dentre os elementos posteriores a ci , i.e., temos n + k + 1 − i candidatos. Logo, k k n+k+1 n+i = |Ci | = . n+1 i=0 i=0 n n n n Problema 7. Mostre que: 1 · +2· + ... + n · = n2n−1 1 2 n ¸˜ ´ Solucao Vamos contar o numero de maneiras de escolhermos algumas criancas para passearem e, al´ m disso, uma delas ¸ e para ganhar um sorvete. Como a crianca que ganhar´ o sorvete certamente estar´ no passeio, podemos comecar escolhendo- ¸ a a ¸ a. Podemos fazer isso de n maneiras. Das criancas que restaram, devemos escolher algum subconjunto para acompanhar a ¸ primeira crianca, podemos fazer isso de 2n−1 maneiras, isso nos d´ o lado direito. Por outro lado, para cada k ≥ 1, podemos ¸ a escolher k criancas ( n ) e, posteriormente, podemos escolher uma delas para ganhar o sorvete de k maneiras. A soma sobre ¸ k todos os k, contar´ todos os conjuntos. a Problema 8. (Putnam 1962) Mostre que: n n r2 = n(n + 1)2n−2 . r=1 r ¸˜ ¸˜ Solucao Vamos construir uma situacao semelhante ao problema anterior, sendo que agora seremos mais bondosos, al´ m e de escolher uma crianca para ganhar um sorvete tamb´ m escolheremos uma crianca para ganhar um caramelo (a crianca ¸ e ¸ ¸ ´ pode ser a mesma). O lado esquerdo conta essas escolhas como no problema anterior. Agora queremos contar o numero de maneiras de primeiro escolhermos as criancas que v˜ o receber as guloseimas e depois aquelas que ir˜ o acompanh´ -las. Se ¸ a a a essas duas criancas s˜ o diferentes, temos n(n − 1)2n−2 escolhas. Se essas criancas s˜ o iguais, temos n2n−1 escolhas. Agora ¸ a ¸ a basta ver que: n(n − 1)2n−2 + n2n−1 = n(n + 1)2n−2 . Exerc´cios Propostos ı 1. Prove as seguntes identidades utilizando argumentos de contagens duplas: n n n (a) + + ... = 2n 0 1 n n n (b) (−1)k =0 k k=0 n m n n−r (c) = . m r r m−r n n−1 n n! (d) k · =n· e com isso conclua que = . k k−1 k k!(n − k)! k n m n+m (e) (Identidade de Vandermonde) = . j=0 j k−j k 2. Considere a sequˆ ncia de Fibonacci definida por F0 = 0, F1 = 1 e Fn+2 = Fn+1 + Fn . Prove que: e (a) O numero de maneiras de cobrirmos um tabuleiro 2 × n, sem sobreposicao, com pecas 1 × 2 e Fn+1 . ´ ¸˜ ¸ ´ (b) Fn+m = Fn−1 Fm + Fn Fm+1 . n n−k+1 (c) k=0 k = Fn+1 . 3. Definimos por Dn o numero de permutacoes de {1, 2 . . . , n} sem pontos fixos (um elemento i e um ponto fixo quando ´ ¸˜ ´ ele ocupa a posicao i na permutacao). Usando o princ´pio da inclus˜ o-exclus˜ o podemos obter: ¸˜ ¸˜ ı a a 1 1 1 Dn = n! 1 − + − . . . + (−1)n . 1! 2! n! Mostre que: n n n! = Dn−r . r=0 r 3
  • 4. ´ 4. Encontre uma formula fechada para a soma 2 2 2 n n n + + ... + . 0 1 n 5. Seja n um inteiro n˜ o negativo. Mostre que: a n 2 n 2n − 1 k =n k n−1 k=0 3 Tabuleiros ` ´ Se o que queremos contar est´ de alguma forma associado a numeros escritos em um tabuleiro, temos uma contagem dupla a ´ ´ ` ´ extremamente natural: a soma dos numeros escritos nas linhas e igual a soma dos numeros escritos nas colunas. ı ¸˜ Problema 9. (Olimp´ada Russa) Duzentos estudantes participaram de uma competicao matem´ tica. A prova possuia 6 a problemas. Sabemos que cada problema foi resolvido corretamente por pelo menos 120 estudantes. Prove que existem dois ¸˜ participantes de modo que para qualquer problema, pelo menos um deles dois conseguiu uma solucao correta. Solucao Facamos um tabuleiro 200 × 6 representando o resultado dos estudantes. Cada linha representar´ um estudante ¸˜ ¸ a e cada problema resolvido pelo estudante i ser´ marcado com o numero 1 na tabela. Caso o problema n˜ o tenha sido re- a ´ a ´ solvido, marcaremos o numero zero. Pensemos inicialmente em casos extremais. O que acontece se um estudante resolveu os seis problemas? Basta escolhermos um estudante qualquer e a dupla desejada estar´ formada. Se um estudante resolveu a 5 problemas, tamb´ m podemos obter facilmente nossa dupla. E se um estudante tiver resolvido exatamente 4 problemas? e Suponha, sem perda de generalidade, que ele n˜ o resolveu os problemas 5 e 6. Sabemos que pelo menos 120 pessoas re- a solveram o problema 5. Se nenhuma delas tiver resolvido o problema 6, saberemos que no m´ ximo 80 pessoas o resolveram. a Mas isso contradiz o enunciado e assim temos certeza que pelo menos uma pessoa resolveu ambos os problemas. Resta mostrar que esse tipo de situacao sempre acontece, i.e., existe algu´ m que resolveu pelo menos 4 problemas. Agora usare- ¸˜ e mos a contagem dupla. A soma dos numero das colunas e pelo menos 6 × 120 = 720. Como existem 200 linhas, pelo menos ´ ´ 720 uma delas ter´ soma 200 > 3, ou seja, pelo menos uma linha ter´ 4 numeros 1’s. a a ´ A estrat´ gia do problema anterior foi associar uma matriz com entradas em {0, 1} que codificasse o enunciado e realizar e ´ ´ as duas somas poss´veis no numero de uns: pelas linhas e pelas colunas. Outra estrat´ gia importante e contar pares de ı e ´ ¸˜ numeros de uns ou zeros em uma mesma linha ou coluna. Vejamos uma nova solucao para o problema anterior usando essa ideia. ¸˜ ¸˜ ´ Solucao Suponha que a afirmacao e falsa, i.e., para cada par de estudantes, existe pelo menos um problema que n˜ o foi a resolvido por eles. Facamos uma matriz 200 × 6 como anteriormente. Para cada par de linhas j e k, cole uma etiqueta ¸ Ejk ao problema i se ele n˜ o foi resolvido pelos estudantes correspondentes. Contemos o numero de etiquetas utilizadas. a ´ Para cada par de linhas, devemos usar pelo menos uma etiqueta, logo o numero m´nimo de etiquetas e 200 . Como cada ´ ı ´ 2 80 problema foi resolvido por pelo menos 120 estudantes, os seis problemas podem receber no m´ ximo 6 · 2 etiquetas. Como a 200 2 < 6 · 80 , temos um absurdo. 2 Problema 10. (Olimp´ada Russa) Bruno pintou k casas de um tabuleiro n × n de preto. Ele observou que n˜ o existiam ı a quatro casas pretas formando um um retˆ ngulo com lados paralelos aos lados do tabuleiro. Mostre que: a √ 1 + 4n − 3 k≤n . 2 ¸˜ Solucao Ao pintar duas casas pretas em uma mesma linha, Bruno cria um empedimento para sua pintura: n˜ o poder´ a a pintar outras duas casas pretas nessas colunas. Comecemos contando esses empedimentos. Denotemos por ai o numero de ´ n casas pintadas na linha i, ent˜ o i=1 ai = k. Para cada par de casas pintadas na linha i, associemos uma etiqueta Ejk se a essas casas est˜ o nas colunas j e k. O numero de etiquetas utilizadas e a ´ ´ n ai , i=1 2 ai pois em cada linha temos 2 pares de quadrados pintados. Como n˜ o podemos repetir etiquetas, pois assim formar´amos a ı 4
  • 5. n ´ a ´ um quadrado, o numero m´ ximo de etiquetas utilizadas e 2 . Logo, n ai n ≤ i=1 2 2 n a2 − ai i n2 − n ≤ . i=1 2 2 n n Pela desigualdade de Cauchy, ( i=1 a2 )n i ≥( i=1 ai )2 = k 2 , consequentemente, k2 k n2 − n − ≤ 2n 2n 2 Estudando o sinal da inequacao do segundo grau em k, obtemos que, ¸˜ √ 1 + 4n − 3 k≤n . 2 Problema 11. (IMO 1998/2) Num concurso, h´ a candidatos e b ju´zes, onde b ≥ 3 e ´mpar. Cada candidato e avaliado por a ı ´ı ´ cada juiz, podendo passar ou n˜ o. Sabe-se que os julgamentos de cada par de ju´zes coincidem em no m´ ximo k candidatos. a ı a Prove que k b−1 ≥ a 2b Solucao Facamos um tabuleiro a × b onde as linhas representam os candidatos e as colunas os ju´zes. Na intersecao de uma ¸˜ ¸ ı ¸˜ linha e coluna, colocaremos um numero 1 caso aquele ju´z tenha aprovado aquele estudante e o numero 0 caso contr´ rio. ´ ı ´ a Se os ju´zes das colunas i e j concordam com o julgamento do aluno da coluna k, pregamos uma etiqueta Eij nesse aluno. ı ´ Calculemos o numero de etiquetas pregadas de duas maneiras. Primeiramente, para cada par de colunas, sabemos que a ´ ´ ` b existem no m´ ximo k etiquetas associadas. Logo, o numero de etiquetas usadas e menor ou igual a 2 k. Para contar esse numero usando as linhas, denotaremos por ai o numero de zeros na linha i. Assim, para cada linha, temos exatamente ´ ´ ai 2 + b−ai . As duas contagens resultam em: 2 a ai b − ai b + ≤ k i=0 2 2 2 Agora vamos fazer outra contagem dupla para estimar o termo ai + b−ai . O que ele conta? Conta o numero de maneira 2 2 ´ de escolhermos duas pessoas do mesmo sexo em um grupo de ai homens e b − ai mulheres. Esse numero tamb´ m pode ´ e b ser calculado contando o numero escolhas de duas pessoas quaisquer ( 2 ) retirando-se aquelas escolhas mistas (ai (b − ai )). ´ Como b e impar, ´ 1 ≤ (b − 2ai )2 1 + 4ai (b − ai ) ≤ b2 b2 − 1 ai (b − ai ) ≤ 4 Substituindo na primeira desigualdade, obtemos: a b b2 − 1 b − ≤ k i=0 2 4 2 b(b − 1) (b − 1)(b + 1) b(b − 1) a −a ≤ k 2 4 2 b+1 ab − ≤ kb 2 b−1 k ≤ 2b a Exerc´cios Propostos ı 1. (Olimp´ada Russa) Com os d´gitos 1 e 2 formamos 5 numeros de n d´gitos de tal forma que dois quaisquer destes ı ı ´ ı numeros coincidam em exatamente m casas decimais e n˜ o existe nenhuma casa decimal onde coincidam os 5 numeros. ´ a ´ Demonstre que: 2 m 3 ≤ ≤ . 5 n 5 5
  • 6. 2. Em um certo comitˆ , cada membro pertence a exatamente trˆ s subcomitˆ s e cada subcomitˆ possui exatamente trˆ s e e e e e ´ ´ ´ membros. Prove que o numero de membros e igual ao numero de subcomitˆ s.e 3. (Olimp´ada Balcˆ nica 1997) Sejam m e n inteiros maiores que 1. Seja S um conjunto com n elementos, e sejam ı a A1 , A2 , . . . , Am suconjuntos de S. Assuma que para quaisquer dois elementos x e y em S, existe um conjunto Ai tal que ou x est´ em Ai e y n˜ o est´ em Ai ou x n˜ o est´ em Ai e y est´ em Ai . Prove que n ≤ 2m . a a a a a a 4. (Olimp´ada Romena 2003) Um inteiro n, n ≥ 2, e bonito se existe uma fam´lia de conjuntos A1 , A2 , . . . , An de subcon- ı ´ ı juntos do conjunto {1, 2, . . . , n} tais que: (1) i ∈ Ai para todo i. (2) i ∈ Aj se e somente se j ∈ Ai , para ´ndices distintos i e j. / ı (3) Ai ∩ Aj e n˜ o vazio para todos i, j ∈ {1, 2, . . . , n}. ´ a Prove que: a) 7 e bonito. ´ b) n e bonito se e somente se n ≥ 7. ´ 4 Comitˆ s e Conjuntos e ´ O caso mais comum para usarmos contagens duplas e quando o conjunto que queremos contar possui uma estrutura de porduto A × B, pois nesse caso: |(a, B)| = |(A, b)| a∈A b∈B ¸˜ Nessa secao, trataremos de alguns exemplos onde esta estruturua n˜ o est´ t˜ o evidente no problema. a a a Problema 12. 1 H´ n tipos de doce na loja do Z´ . Um dia, m amigos se juntam para comprarem k doces diferentes cada um. Cada tipo de a e doce e comprado por r dos amigos. Cada par de tipo de doces e escolhido por t amigos. Prove que: ´ ´ (i) mk = nr; (ii) r(k − 1) = t(n − 1). Solucao O numero de doces levados da loja e mk. Como cada doce e levado por r amigos, esse numero tamb´ m e nr. ¸˜ ´ ´ ´ ´ e ´ a ¸˜ ´ ´ r Quantos pares de doces s˜ o levados? Pela ultima informacao do enunciado, esse numero e t 2 . Por outro lado, cada amigo ´ leva k pares de doces, logo: 2 r k t = m 2 2 r(r − 1) k(k − 1) t = m 2 2 r(r − 1) nr(k − 1) t = 2 2 r(k − 1) = t(n − 1) Problema 13. 2 Seja X um conjunto com n elementos (n ≥ 1). Suponha que F = {A1 , A2 , . . . , Am } e uma fam´lia de ´ ı subconjuntos de X com a propriedade que |Ai ∩ Aj | = 1, para todos i = j. Mostre que m ≤ n. Solucao Suponhamos inicialmente que m ≥ 2 (Quando m < 2 o problema e trivial). Al´ m disso, suponhamos que nenhum ¸˜ ´ e Ai e vazio ou igual ao conjunto todo. Iremos nos concentrar nos pares (x, A) onde x ∈ A. Suponha que o elemento x pertence ´ / exatamente aos d(x) conjuntos: {A1 , A2 , . . . , Ad(x) }. Como |A ∩ Ai | = 1, para cada i ∈ {1, 2, . . . , d(x)}, deve existir um xi = x 1 Vocˆ e poder´ encontrar mais exemplos como esse estudando Block designs. a 2 Este ´ ¸˜ ´ problema e um caso particular da desigualdade de Fischer. Uma solucao elementar para este problema pode ser encontrada usando Algebra Linear 6
  • 7. tal que xi ∈ A ∩ Ai . Al´ m disso, se i = j, xi = xj e consequentemente |A| ≥ d(x). Se m > n, m − d(x) > n − d(x) ≥ n − |A| e e portanto, d(x) |A| < . m − d(x) n − |A| |A| Para cada tal par (x, A), some o numero ´ . Qual numeros iremos obter? Fixado A, existem n − |A| poss´veis valores ´ ı n − |A| de x, logo: |A| (n − |A|)|A| = = |A|. n − |A| n − |A| (x,A) A A Fixado x, existem m − d(x) possibilidade para A, portanto, a soma anterior deve ser maior que d(x) (m − d(x)d(x)) = = d(x). m − d(x) x m − d(x) x (x,A) Assim obtivemos um absurdo pois A |A| = x d(x) (estamos contando os elementos que foram usados nos conjuntos de duas formas). Os casos em que algum dos Ai ’s e igual ao conjunto todo ou o conjunto vazio e deixado para o leitor. ´ ´ Problema 14. (Banco IMO 2004) Existem n estudantes em uma universidade, n inteiro ´mpar. Alguns estudantes se reunem ı em clubes (um estudante pode pertencer a clubes diferentes). Alguns clubes se reunem para formar algumas sociedades(um clube pode pertencer a diferentes sociedades). Existem k sociedades. Suponham que acontecam as seguintes condicoes: ¸ ¸˜ i) cada par de estudantes est´ em exatamente um clube. a ii) para cada estudante e cada sociedade, o estudante est´ em exatamente um clube da sociedade, a iii) cada clube tem um numero ´mpar de estudantes e, al´ m disso, um clube com 2m + 1 estudantes (m > 0) est´ em ´ ı e a exatamente m sociedades. Encontre todos os valores poss´veis de k. ı Solucao Vamos contar o numero de triplas (e, C, S) onde e e um estudante, C e um clube e S e uma sociedade. Fixados ¸˜ ´ ´ ´ ´ e e S, existe apenas uma possibilidade para C, assim, em nossa primeira contagem obtivemos nk tais triplas. Fixado uma |C| − 1 comiss˜ o C, temos a possibilidades para S, e uma vez que escolhemos C e S, temos |C| possibilidades para e(ele deve 2 ser um estudante em C). Assim, o numero de tais triplas e ´ ´ |C|(|C| − 1) nk = 2 C = total de pares de estudantes nos clubes n = 2 n−1 E consequentemente, k = ´ ´ . Um exemplo onde este valor e atigindo e considerando apenas um clube com todos os 2 estudantes e k sociedades iguais formadas por esse unico clube. ´ Exerc´cios Propostos ı ˜ 1. Nove cientistas trabalham em um projeto sigiloso. Por questoes de seguranca, os planos s˜ o guardados em um cofre ¸ a ´ ´ protegido por muitos cadeados de modo que so e poss´vel abr´-los todos se houver pelo menos 5 cientistas presentes. ı ı ´ ´ (i) Qual e o numero m´nimo poss´vel de cadeados? ı ı (ii) Na situacao do item (i), quantas chaves cada cientista deve ter? ¸˜ 2. (Olimp´ada Russa) ı (i) Uma comiss˜ o se reuniu 40 vezes. Em cada reuni˜ o estiveram presentes 10 pessoas de tal maneira que quaisquer a a dois dos membros da comiss˜ o n˜ o estiveram juntos em mais de uma oportunidade. Demonstre que a quantidade a a a ´ de membros da comiss˜ o e maior que 60. (ii) Demonstre que com 25 pessoas n˜ o se pode formar mais que 30 comissoes de 5 pessoas cada uma de modo que a ˜ ˜ n˜ o existam duas comissoes que tenham mais de um membro em comum. a 7
  • 8. 3. (OBM,1992/8) Em um torneio de xadrez cada jogador disputou uma partida com cada um dos demais participantes. A 1 cada partida, havendo empate, cada jogador ganhou ponto; caso contr´ rio, o vencedor ganhou 1 ponto e o perdedor a 2 ´ 0 pontos. Participaram homens e mulheres e cada participante conquistou o mesmo numero de pontos contra homens ´ ´ que contra mulheres. Mostre que o numero de participantes e um quadrado perfeito. 5 Problemas Suplementares ¸˜ Os problemas desta secao exigem mais engenhosidade. Teorema 1. (Erdos-Ko-Rado) Seja X um conjunto com n elementos e F = {A1 , A2 , . . . , Ap } uma fam´lia de subconjuntos de ı X que satisfazem as seguintes condicoes ¸˜ n 1. |Ai | = k ≤ para todo i. 2 2. Ai ∩ Aj = ∅ para todos i e j distintos. n−1 Ent˜ o p ≤ a k−1 . Prova.Comecemos com um lema que aparentemente n˜ o apresenta nenhuma conex˜ o com o problema: a a Lema. Considere um c´rculo C dividido por n pontos. Um arco de tamanho k consiste de k + 1 pontos consecutivos e k ı arestas entre eles. Seja n ≥ 2k, e suponha que existem t arcos distintos A1 , A2 , . . . , At , todos de tamanho k, tais que quaisquer dois arcos possuem uma aresta em comum. Ent˜ o t ≤ k. a A prova do lema ser´ deixada como exerc´cio para o leitor. Seja F uma fam´lia como no enunciado do teorema. Cada a ı ı permutacao c´clica dos elementos de X corresponde a uma divis˜ o em pontos e arcos, onde cada elemento do conjunto ¸˜ ı a est´ entre dois pontos. Cada conjunto de k elementos pode ser visto como um arco de tamanho k formando por elementos a justapostos (em uma ordem qualquer). Para cada tal permutacao, podemos calcular quantos arcos de tamanho k correspon- ¸˜ dem a conjuntos da fam´lia. Pelo lema, esse numero e menor ou igual a k. Como existem (n − 1)! permutacoes, a quantidade ı ´ ´ ` ¸˜ m´ xima de conjuntos que podem ser encontrados nessas permutacoes e k(n − 1)!. Certamente nessa contagem, alguns con- a ¸˜ ´ ´ juntos foram contados v´ rias vezes. E agora que usaremos uma contagem dupla. Fixado um conjunto A, seus elementos a podem estar arranjandos para formarem um arco de tamanho k de k! maneiras. Os outros elementos do conjunto podem ser permutados nas posicoes que restaram de (n − k)! maneiras. Ou seja, cada conjunto e contado exatamente k!(n − k)! vezes. ¸˜ ´ Como existem p conjuntos, temos: pk!(n − k)! ≤ k(n − 1)!; n−1 p ≤ k−1 Problema 15. (Olimp´ada do Leningrado 1987) Sejam A1 , A2 , . . . , As uma fam´lia de subconjuntos de {1, 2, . . . , n} tais que ı ı |Ai | = ai . Sabemos que nenhum desses subconjuntos cont´ m outro subconjunto da fam´lia. Prove que e ı −1 −1 −1 n n n + + ... + ≤1 a1 a2 as ¸˜ Problema 16. (IMO 2005) Numa competicao de matem´ tica na qual foram propostos 6 problemas, quaisquer dois prob- a lemas foram resolvidos por mais de 2/5 dos estudantes. Al´ m disso, nenhum estudante resolveu todos os 6 problemas. e Mostre que existem pelo menos 2 estudantes que resolveram 5 problemas cada um. Problema 17. Alguns asteriscos est˜ o escritos nas casas de um tabuleiro m × n(m < n), de modo que existe pelo menos a um asterisco em cada coluna. Mostre que que existe um asterisco A tal que lA > cA , onde la e cA denotam as quantidade de asteriscos na linha e coluna de A, respectivamente. Problema 18. (Olimp´ada Iberoamericana 2001) Sejam S um conjunto de n elementos e S1 , S2 , . . . , Sk subconjuntos de ı S(k ≥ 2), tais que cada um deles tˆ m pelo menos r elementos. Demonstre que existem i e j, com i = j, tais que a quantidade e nk de elementos em comum entre Si e Sj e maior ou igual a r − ´ ` . 4(k − 1) 8
  • 9. Problema 19. (IMO 1989) Sejam n e k dois inteiros positivos e seja S um conjunto de n pontos num plano tais que 1. n˜ o haja trˆ s pontos de S que sejam colineares; a e 2. para qualquer ponto P de S, h´ pelo menos k pontos de S que s˜ o equidistantes de P . a a √ Prove que k < 1/2 + 2n. Problema 20. (IMO 1987) 3 Seja pn (k) o total de permutacoes do conjunto {1, 2, 3, . . . , n} que contˆ m exatamente k pontos ¸˜ e fixos. Prove que n k · pn (k) = n!. k=1 Observacao: uma permutacao f de um conjunto S e uma funcao bijetora de S em S; um elemento i de S e chamado ponto ¸˜ ¸˜ ´ ¸˜ ´ fixo da permutacao f se f (i) = i. ¸˜ 3 O enunciado mais geral desse problema e o seguinte: Seja G um grupo finito agindo sobre um conjuno M . Sejam k o numero de orbitas em que M e ´ ´ ´ ´ dividido por esta acao e F ix(g), g ∈ G, o numero de pontos fixos de g. Ent˜ o: ¸˜ ´ a F ix(g) = k · |G|. g∈G ´ Esse e o lema de Burnside. 9